Difference between revisions of "2024 AMC 8 Problems/Problem 16"

(Created page with "Four pigs are piggling. Mommy pig also wants to piggle. How many ways can mommy piggle?")
 
(Solution 4)
 
(46 intermediate revisions by 23 users not shown)
Line 1: Line 1:
Four pigs are piggling. Mommy pig also wants to piggle. How many ways can mommy piggle?
+
==Problem 16 ==
 +
 
 +
Minh enters the numbers <math>1</math> through <math>81</math> into the cells of a <math>9 \times 9</math> grid in some order. She calculates the product of the numbers in each row and column. What is the least number of rows and columns that could have a product divisible by <math>3</math>?
 +
 
 +
<math>\textbf{(A) } 8\qquad\textbf{(B) } 9\qquad\textbf{(C) } 10\qquad\textbf{(D) } 11\qquad\textbf{(E) } 12</math>
 +
 
 +
==Solution==
 +
 
 +
<asy>
 +
unitsize(0.2cm);
 +
draw((9,18)--(-9,18));
 +
draw((9,16)--(-9,16));
 +
draw((9,14)--(-9,14));
 +
draw((9,12)--(-9,12));
 +
draw((9,10)--(-9,10));
 +
draw((9,8)--(-9,8));
 +
draw((9,6)--(-9,6));
 +
draw((9,4)--(-9,4));
 +
draw((9,2)--(-9,2));
 +
draw((9,0)--(-9,0));
 +
 
 +
draw((9,18)--(9,0));
 +
draw((7,18)--(7,0));
 +
draw((5,18)--(5,0));
 +
draw((3,18)--(3,0));
 +
draw((1,18)--(1,0));
 +
draw((-1,18)--(-1,0));
 +
draw((-3,18)--(-3,0));
 +
draw((-5,18)--(-5,0));
 +
draw((-7,18)--(-7,0));
 +
draw((-9,18)--(-9,0));
 +
 
 +
draw((-9,17)--(9,17), red);
 +
draw((-9,15)--(9,15), red);
 +
draw((-9,13)--(9,13), red);
 +
draw((-8,0)--(-8,18), red);
 +
draw((-6,0)--(-6,18), red);
 +
draw((-4,0)--(-4,18), red);
 +
draw((-2,0)--(-2,18), red);
 +
</asy>
 +
 
 +
We know that if a row/column of numbers has a single multiple of <math>3</math>, that entire row/column will be divisible by <math>3</math>. Since there are <math>27</math> multiples of <math>3</math> from <math>1</math> to <math>81</math>, We need to find a way to place the <math>54</math> non-multiples of <math>3</math> such that they take up as many entire rows and columns as possible.
 +
If we naively put in non-multiples of <math>3</math> in <math>6</math> rows from the top, we get <math>18 - 6 = 12</math> rows that are multiples of <math>3</math>. However, we can improve this number by making some rows and columns intersect so that some squares help fill out both rows and columns
 +
We see that filling <math>7</math> rows/columns would usually take <math>7 \times 9 = 63</math> of our non-multiples, but if we do <math>4</math> rows and <math>3</math> columns, <math>12</math> will intersect. With our <math>54</math> being enough as we need only <math>51</math> non-multiples of <math>3</math>(<math>63</math> minus the <math>12</math> overlapped). We check to see if we can fill out one more row/column, and when that fails we conclude the final answer to be <math>18 - 7 = \boxed{\textbf{(D)} 11}</math>  -IwOwOwl253 ~andliu766(Minor edits) -c29ss1(Diagram)
 +
 
 +
==Solution 2==
 +
Note you can swap/rotate any configuration of rows, such that all the rows and columns that have a product of 3 are in the top left. Hence the points are bounded by a <math>a \times b</math> rectangle. This has <math>ab</math> area and <math>a+b</math> rows and columns divisible by <math>3</math>. We want <math>ab\ge 27</math> and <math>a+b</math> minimized.
 +
 
 +
If <math>ab=27</math>, we achieve minimum with <math>a+b=9+3=12</math>.
 +
 
 +
If <math>ab=28</math>,our best is <math>a+b=7+4=11</math>. Note if <math>a+b=10</math>, then <math>ab\le 25</math>, and hence there is no smaller answer, and we get <math>\boxed{\textbf{(D)} 11}</math>.
 +
 
 +
- SahanWijetunga
 +
~vockey(minor edits)
 +
 
 +
==Solution 3==
 +
 
 +
For a row or column to have a product divisible by <math>3</math>, there must be a multiple of <math>3</math> in the row or column. To create the least amount of rows and columns with multiples of <math>3</math>, we must find a way to keep them all together, to minimize the total number of rows and columns. From <math>1</math> to <math>81</math>, there are <math>27</math> multiples of <math>3</math> (<math>81/3</math>). So we have to fill <math>27</math> cells with numbers that are multiples of <math>3</math>. If we put <math>25</math> of these numbers in a <math>5 x 5</math> grid, there would be <math>5</math> rows and <math>5</math> columns (<math>10</math> in total), with products divisible by <math>3</math>. However, we have <math>27</math> numbers, so <math>2</math> numbers remain to put in the <math>9 x 9</math> grid. If we put both numbers in the <math>6</math>th column, but one in the first row, and one in the second row, (next to the <math>5 x 5</math> already filled), we would have a total of <math>6</math> columns now, and still <math>5</math> rows with products that are multiples of <math>3</math>. So the answer is <math>\boxed{\textbf{(D)} 11}</math>
 +
 
 +
~goofytaipan
 +
 
 +
==Solution 4==
 +
 
 +
In the numbers <math>1</math> to <math>81</math>, there are 27 multiples of three. In order to minimize the rows and columns, the best way is to make a square. However, the closest square is <math>25</math>, meaning there are two multiples of three remaining. However, you can place these multiples right above the 5x5 square, meaning the answer is <math>\boxed{\textbf {(D)} 11}</math>
 +
~ e___
 +
 
 +
==Video Solution by Math-X (Apply this simple strategy that works every time!!!)==
 +
https://youtu.be/BaE00H2SHQM?si=Z4Y7xHZEdRfDR-Bb&t=3952
 +
 
 +
~Math-X
 +
==Video Solution 1 (easy to digest) by Power Solve==
 +
https://youtu.be/zxkL4c316vg
 +
 
 +
==Video Solution 2 by OmegaLearn.org==
 +
https://youtu.be/xfiPVmuMiXs
 +
 
 +
==Video Solution 3 by SpreadTheMathLove==
 +
https://www.youtube.com/watch?v=Svibu3nKB7E
 +
 
 +
==Video Solution by NiuniuMaths (Easy to understand!)==
 +
https://www.youtube.com/watch?v=V-xN8Njd_Lc
 +
 
 +
~NiuniuMaths
 +
 
 +
== Video Solution by CosineMethod [🔥Fast and Easy🔥]==
 +
 
 +
https://www.youtube.com/watch?v=DLzFB4EplKk
 +
==Video Solution by Interstigation==
 +
https://youtu.be/ktzijuZtDas&t=1709
 +
 
 +
==See Also==
 +
{{AMC8 box|year=2024|num-b=15|num-a=17}}
 +
{{MAA Notice}}

Latest revision as of 14:26, 19 February 2024

Problem 16

Minh enters the numbers $1$ through $81$ into the cells of a $9 \times 9$ grid in some order. She calculates the product of the numbers in each row and column. What is the least number of rows and columns that could have a product divisible by $3$?

$\textbf{(A) } 8\qquad\textbf{(B) } 9\qquad\textbf{(C) } 10\qquad\textbf{(D) } 11\qquad\textbf{(E) } 12$

Solution

[asy] unitsize(0.2cm); draw((9,18)--(-9,18)); draw((9,16)--(-9,16)); draw((9,14)--(-9,14)); draw((9,12)--(-9,12)); draw((9,10)--(-9,10)); draw((9,8)--(-9,8)); draw((9,6)--(-9,6)); draw((9,4)--(-9,4)); draw((9,2)--(-9,2)); draw((9,0)--(-9,0));  draw((9,18)--(9,0)); draw((7,18)--(7,0)); draw((5,18)--(5,0)); draw((3,18)--(3,0)); draw((1,18)--(1,0)); draw((-1,18)--(-1,0)); draw((-3,18)--(-3,0)); draw((-5,18)--(-5,0)); draw((-7,18)--(-7,0)); draw((-9,18)--(-9,0));  draw((-9,17)--(9,17), red); draw((-9,15)--(9,15), red); draw((-9,13)--(9,13), red); draw((-8,0)--(-8,18), red); draw((-6,0)--(-6,18), red); draw((-4,0)--(-4,18), red); draw((-2,0)--(-2,18), red); [/asy]

We know that if a row/column of numbers has a single multiple of $3$, that entire row/column will be divisible by $3$. Since there are $27$ multiples of $3$ from $1$ to $81$, We need to find a way to place the $54$ non-multiples of $3$ such that they take up as many entire rows and columns as possible. If we naively put in non-multiples of $3$ in $6$ rows from the top, we get $18 - 6 = 12$ rows that are multiples of $3$. However, we can improve this number by making some rows and columns intersect so that some squares help fill out both rows and columns We see that filling $7$ rows/columns would usually take $7 \times 9 = 63$ of our non-multiples, but if we do $4$ rows and $3$ columns, $12$ will intersect. With our $54$ being enough as we need only $51$ non-multiples of $3$($63$ minus the $12$ overlapped). We check to see if we can fill out one more row/column, and when that fails we conclude the final answer to be $18 - 7 = \boxed{\textbf{(D)} 11}$ -IwOwOwl253 ~andliu766(Minor edits) -c29ss1(Diagram)

Solution 2

Note you can swap/rotate any configuration of rows, such that all the rows and columns that have a product of 3 are in the top left. Hence the points are bounded by a $a \times b$ rectangle. This has $ab$ area and $a+b$ rows and columns divisible by $3$. We want $ab\ge 27$ and $a+b$ minimized.

If $ab=27$, we achieve minimum with $a+b=9+3=12$.

If $ab=28$,our best is $a+b=7+4=11$. Note if $a+b=10$, then $ab\le 25$, and hence there is no smaller answer, and we get $\boxed{\textbf{(D)} 11}$.

- SahanWijetunga ~vockey(minor edits)

Solution 3

For a row or column to have a product divisible by $3$, there must be a multiple of $3$ in the row or column. To create the least amount of rows and columns with multiples of $3$, we must find a way to keep them all together, to minimize the total number of rows and columns. From $1$ to $81$, there are $27$ multiples of $3$ ($81/3$). So we have to fill $27$ cells with numbers that are multiples of $3$. If we put $25$ of these numbers in a $5 x 5$ grid, there would be $5$ rows and $5$ columns ($10$ in total), with products divisible by $3$. However, we have $27$ numbers, so $2$ numbers remain to put in the $9 x 9$ grid. If we put both numbers in the $6$th column, but one in the first row, and one in the second row, (next to the $5 x 5$ already filled), we would have a total of $6$ columns now, and still $5$ rows with products that are multiples of $3$. So the answer is $\boxed{\textbf{(D)} 11}$

~goofytaipan

Solution 4

In the numbers $1$ to $81$, there are 27 multiples of three. In order to minimize the rows and columns, the best way is to make a square. However, the closest square is $25$, meaning there are two multiples of three remaining. However, you can place these multiples right above the 5x5 square, meaning the answer is $\boxed{\textbf {(D)} 11}$ ~ e___

Video Solution by Math-X (Apply this simple strategy that works every time!!!)

https://youtu.be/BaE00H2SHQM?si=Z4Y7xHZEdRfDR-Bb&t=3952

~Math-X

Video Solution 1 (easy to digest) by Power Solve

https://youtu.be/zxkL4c316vg

Video Solution 2 by OmegaLearn.org

https://youtu.be/xfiPVmuMiXs

Video Solution 3 by SpreadTheMathLove

https://www.youtube.com/watch?v=Svibu3nKB7E

Video Solution by NiuniuMaths (Easy to understand!)

https://www.youtube.com/watch?v=V-xN8Njd_Lc

~NiuniuMaths

Video Solution by CosineMethod [🔥Fast and Easy🔥]

https://www.youtube.com/watch?v=DLzFB4EplKk

Video Solution by Interstigation

https://youtu.be/ktzijuZtDas&t=1709

See Also

2024 AMC 8 (ProblemsAnswer KeyResources)
Preceded by
Problem 15
Followed by
Problem 17
1 2 3 4 5 6 7 8 9 10 11 12 13 14 15 16 17 18 19 20 21 22 23 24 25
All AJHSME/AMC 8 Problems and Solutions

The problems on this page are copyrighted by the Mathematical Association of America's American Mathematics Competitions. AMC logo.png